Difference between revisions of "Thread:User talk:Dsekercioglu/MEA/And to make it even faster/reply (10)"

From Robowiki
Jump to navigation Jump to search
m
m
 
Line 1: Line 1:
 
I also agree that it's wrong. But why is it bigger than the correct one? Can you clarify that?
 
I also agree that it's wrong. But why is it bigger than the correct one? Can you clarify that?
  
I understood that the problem with this is that it assumes that it's optimal to move in a fixed retreat angle (like moving in a straight line), when it's actually true that the best retreat angle isn't a function only of the initial position, but it changes as you move around the wave. Is that the real issue?
+
I understood that the problem with this is that it assumes that it's optimal to move in a fixed retreat angle (like moving in a perfect orbit), when it's actually true that the best retreat angle isn't a function only of the initial position, but it changes as you move around the wave. Is that the real issue?

Latest revision as of 13:04, 24 September 2017

I also agree that it's wrong. But why is it bigger than the correct one? Can you clarify that?

I understood that the problem with this is that it assumes that it's optimal to move in a fixed retreat angle (like moving in a perfect orbit), when it's actually true that the best retreat angle isn't a function only of the initial position, but it changes as you move around the wave. Is that the real issue?